IndexError: list assignment index out of range in Python

avatar

Last updated: Apr 8, 2024 Reading time · 9 min

banner

# Table of Contents

  • IndexError: list assignment index out of range
  • (CSV) IndexError: list index out of range
  • sys.argv[1] IndexError: list index out of range
  • IndexError: pop index out of range
Make sure to click on the correct subheading depending on your error message.

# IndexError: list assignment index out of range in Python

The Python "IndexError: list assignment index out of range" occurs when we try to assign a value at an index that doesn't exist in the list.

To solve the error, use the append() method to add an item to the end of the list, e.g. my_list.append('b') .

indexerror list assignment index out of range

Here is an example of how the error occurs.

assignment to index out of range

The list has a length of 3 . Since indexes in Python are zero-based, the first index in the list is 0 , and the last is 2 .

abc
012

Trying to assign a value to any positive index outside the range of 0-2 would cause the IndexError .

# Adding an item to the end of the list with append()

If you need to add an item to the end of a list, use the list.append() method instead.

adding an item to end of list with append

The list.append() method adds an item to the end of the list.

The method returns None as it mutates the original list.

# Changing the value of the element at the last index in the list

If you meant to change the value of the last index in the list, use -1 .

change value of element at last index in list

When the index starts with a minus, we start counting backward from the end of the list.

# Declaring a list that contains N elements and updating a certain index

Alternatively, you can declare a list that contains N elements with None values.

The item you specify in the list will be contained N times in the new list the operation returns.

Make sure to wrap the value you want to repeat in a list.

If the list contains a value at the specific index, then you are able to change it.

# Using a try/except statement to handle the error

If you need to handle the error if the specified list index doesn't exist, use a try/except statement.

The list in the example has 3 elements, so its last element has an index of 2 .

We wrapped the assignment in a try/except block, so the IndexError is handled by the except block.

You can also use a pass statement in the except block if you need to ignore the error.

The pass statement does nothing and is used when a statement is required syntactically but the program requires no action.

# Getting the length of a list

If you need to get the length of the list, use the len() function.

The len() function returns the length (the number of items) of an object.

The argument the function takes may be a sequence (a string, tuple, list, range or bytes) or a collection (a dictionary, set, or frozen set).

If you need to check if an index exists before assigning a value, use an if statement.

This means that you can check if the list's length is greater than the index you are trying to assign to.

# Trying to assign a value to an empty list at a specific index

Note that if you try to assign to an empty list at a specific index, you'd always get an IndexError .

You should print the list you are trying to access and its length to make sure the variable stores what you expect.

# Use the extend() method to add multiple items to the end of a list

If you need to add multiple items to the end of a list, use the extend() method.

The list.extend method takes an iterable (such as a list) and extends the list by appending all of the items from the iterable.

The list.extend method returns None as it mutates the original list.

# (CSV) IndexError: list index out of range in Python

The Python CSV "IndexError: list index out of range" occurs when we try to access a list at an index out of range, e.g. an empty row in a CSV file.

To solve the error, check if the row isn't empty before accessing it at an index, or check if the index exists in the list.

csv indexerror list index out of range

Assume we have the following CSV file.

And we are trying to read it as follows.

# Check if the list contains elements before accessing it

One way to solve the error is to check if the list contains any elements before accessing it at an index.

The if statement checks if the list is truthy on each iteration.

All values that are not truthy are considered falsy. The falsy values in Python are:

  • constants defined to be falsy: None and False .
  • 0 (zero) of any numeric type
  • empty sequences and collections: "" (empty string), () (empty tuple), [] (empty list), {} (empty dictionary), set() (empty set), range(0) (empty range).

# Check if the index you are trying to access exists in the list

Alternatively, you can check whether the specific index you are trying to access exists in the list.

This means that you can check if the list's length is greater than the index you are trying to access.

# Use a try/except statement to handle the error

Alternatively, you can use a try/except block to handle the error.

We try to access the list of the current iteration at index 1 , and if an IndexError is raised, we can handle it in the except block or continue to the next iteration.

# sys.argv [1] IndexError: list index out of range in Python

The sys.argv "IndexError: list index out of range in Python" occurs when we run a Python script without specifying values for the required command line arguments.

To solve the error, provide values for the required arguments, e.g. python main.py first second .

sys argv indexerror list index out of range

I ran the script with python main.py .

The sys.argv list contains the command line arguments that were passed to the Python script.

# Provide all of the required command line arguments

To solve the error, make sure to provide all of the required command line arguments when running the script, e.g. python main.py first second .

Notice that the first item in the list is always the name of the script.

It is operating system dependent if this is the full pathname or not.

# Check if the sys.argv list contains the index

If you don't have to always specify all of the command line arguments that your script tries to access, use an if statement to check if the sys.argv list contains the index that you are trying to access.

I ran the script as python main.py without providing any command line arguments, so the condition wasn't met and the else block ran.

We tried accessing the list item at index 1 which raised an IndexError exception.

You can handle the error or use the pass keyword in the except block.

# IndexError: pop index out of range in Python

The Python "IndexError: pop index out of range" occurs when we pass an index that doesn't exist in the list to the pop() method.

To solve the error, pass an index that exists to the method or call the pop() method without arguments to remove the last item from the list.

indexerror pop index out of range

The list has a length of 3 . Since indexes in Python are zero-based, the first item in the list has an index of 0 , and the last an index of 2 .

If you need to remove the last item in the list, call the method without passing it an index.

The list.pop method removes the item at the given position in the list and returns it.

You can also use negative indices to count backward, e.g. my_list.pop(-1) removes the last item of the list, and my_list.pop(-2) removes the second-to-last item.

Alternatively, you can check if an item at the specified index exists before passing it to pop() .

This means that you can check if the list's length is greater than the index you are passing to pop() .

An alternative approach to handle the error is to use a try/except block.

If calling the pop() method with the provided index raises an IndexError , the except block is run, where we can handle the error or use the pass keyword to ignore it.

# Additional Resources

You can learn more about the related topics by checking out the following tutorials:

  • IndexError: index 0 is out of bounds for axis 0 with size 0
  • IndexError: invalid index to scalar variable in Python
  • IndexError: pop from empty list in Python [Solved]
  • Replacement index 1 out of range for positional args tuple
  • IndexError: too many indices for array in Python [Solved]
  • IndexError: tuple index out of range in Python [Solved]

book cover

Borislav Hadzhiev

Web Developer

buy me a coffee

Copyright © 2024 Borislav Hadzhiev

How to fix IndexError: list assignment index out of range in Python

list assignment is out of range

This tutorial will show you an example that causes this error and how to fix it in practice

How to reproduce this error

Because the list has two items, the index number ranges from 0 to 1. Assigning a value to any other index number will cause this error.

How to fix this error

Take your skills to the next level ⚡️.

How to Fix the “List index out of range” Error in Python

Author's photo

  • learn python

At many points in your Python programming career, you’re going to run into the “List index out of range” error while writing your programs. What does this mean, and how do we fix this error? We’ll answer that question in this article.

The short answer is: this error occurs when you’re trying to access an item outside of your list’s range. The long answer, on the other hand, is much more interesting. To get there, we’ll learn a lot about how lists work, how to index things the bad way and the good way, and finally how to solve the above-mentioned error.

This article is aimed at Python beginners who have little experience in programming. Understanding this error early will save you plenty of time down the road. If you’re looking for some learning material, our Python Basics track includes 3 interactive courses bundled together to get you on your feet.

Indexing Python Lists

Lists are one of the most useful data structures in Python. And they come with a whole bunch of useful methods . Other Python data structures include tuples, arrays, dictionaries, and sets, but we won’t go into their details here. For hands-on experience with these structures, we have a Python Data Structures in Practice course which is suitable for beginners.

A list can be created as follows:

Instead of using square brackets ([]) to define your list, you can also use the list() built-in function.

There are already a few interesting things to note about the above example. First, you can store any data type in a list, such as an integer, string, floating-point number, or even another list. Second, the elements don’t have to be unique: the integer 1 appears twice in the above example.

The elements in a list are indexed starting from 0. Therefore, to access the first element, do the following:

Our list contains 6 elements, which you can get using the len() built-in function. To access the last element of the list, you might naively try to do the following:

This is equivalent to print(x[len(x)]) . Since list indexing starts from 0, the last element has index len(x)–1 . When we try to access the index len(x) , we are outside the range of the list and get the error. A more robust way to get the final element of the list looks like this:

While this works, it’s not the most pythonic way. A better method exploits a nice feature of lists – namely, that they can be indexed from the end of the list by using a negative number as the index. The final element can be printed as follows:

The second last element can be accessed with the index -2, and so on. This means using the index -6 will get back to the first element. Taking it one step further:

Notice this asymmetry. The first error was trying to access the element after the last with the index 6, and the second error was trying to access the element before the first with the index -7. This is due to forward indexing starting at 0 (the start of the list), and backwards indexing starting at -1 (the end of the list). This is shown graphically below:

list index out of range

Looping Through Lists

Whenever you’re working with lists, you’ll need to know about loops. A loop allows you to iterate through all the elements in a list.

The first type of loop we’ll take a look at is the while loop. You have to be a little more careful with while loops, because a small mistake will make them run forever, requiring you to force the program to quit. Once again, let’s try to naively loop through our list:

In this example we define our index, i , to start from zero. After every iteration of our while loop, we print the list element and then go to the next index with the += assignment operator. (This is a neat little trick, which is like doing i=i+1 .)

By the way, if you forget the final line, you’ll get an infinite loop.

We encountered the index error for the same reason as in the first section – the final element has index len(x)-1 . Just modify the condition of the while statement to reflect this, and it will work without problems.

Most of your looping will be done with a for loop, which we’ll now turn our attention to. A better method to loop through the elements in our list without the risk of running into the index error is to take advantage of the range() built-in function. This takes three arguments, of which only the stop argument is required. Try the following:

The combination of the range() and len() built-in functions takes care of worrying about when to stop indexing our list to avoid the index out of range error entirely. This method, however, is only useful if you care about knowing what the index is.

For example, maybe you want to print out the index and the element. In that case, all you need to do is modify the print() statement to print(i, x[i]) . Try doing this for yourself to see the result. Alternatively, you can use The enumerate() function in Python.

If you just want to get the element, there’s a simpler way that’s much more intuitive and readable. Just loop through the elements of the list directly:

If the user inputs an index outside the range of the list (e.g. 6), they’ll run into the list index error again. We can modify the function to check the input value with an if statement:

Doing this prevents our program from crashing if the index is out of range. You can even use a negative index in the above function.

There are other ways to do error handling in Python that will help you avoid errors like “list index out of range”. For example, you could implement a try-exceptaa block instead of the if-else statement.

To see a try-except block in action, let’s handle a potential index error in the get_value() function we wrote above. Preventing the error looks like this:

As you can probably see, the second method is a little more concise and readable. It’s also less error-prone than explicitly checking the input index with an if-else statement.

Master the “List index out of range” Error in Python

You should now know what the index out of range error in Python means, why it pops up, and how to prevent it in your Python programs.

A useful way to debug this error and understand how your programs are running is simply to print the index and compare it to the length of your list.

This error could also occur when iterating over other data structures, such as arrays, tuples, or even when iterating through a string. Using strings is a little different from  using lists; if you want to learn the tools to master this topic, consider taking our Working with Strings in Python course. The skills you learnt here should be applicable to many common use cases.

You may also like

list assignment is out of range

How Do You Write a SELECT Statement in SQL?

list assignment is out of range

What Is a Foreign Key in SQL?

list assignment is out of range

Enumerate and Explain All the Basic Elements of an SQL Query

list assignment is out of range

Explore your training options in 10 minutes Get Started

  • Graduate Stories
  • Partner Spotlights
  • Bootcamp Prep
  • Bootcamp Admissions
  • University Bootcamps
  • Coding Tools
  • Software Engineering
  • Web Development
  • Data Science
  • Tech Guides
  • Tech Resources
  • Career Advice
  • Online Learning
  • Internships
  • Apprenticeships
  • Tech Salaries
  • Associate Degree
  • Bachelor's Degree
  • Master's Degree
  • University Admissions
  • Best Schools
  • Certifications
  • Bootcamp Financing
  • Higher Ed Financing
  • Scholarships
  • Financial Aid
  • Best Coding Bootcamps
  • Best Online Bootcamps
  • Best Web Design Bootcamps
  • Best Data Science Bootcamps
  • Best Technology Sales Bootcamps
  • Best Data Analytics Bootcamps
  • Best Cybersecurity Bootcamps
  • Best Digital Marketing Bootcamps
  • Los Angeles
  • San Francisco
  • Browse All Locations
  • Digital Marketing
  • Machine Learning
  • See All Subjects
  • Bootcamps 101
  • Full-Stack Development
  • Career Changes
  • View all Career Discussions
  • Mobile App Development
  • Cybersecurity
  • Product Management
  • UX/UI Design
  • What is a Coding Bootcamp?
  • Are Coding Bootcamps Worth It?
  • How to Choose a Coding Bootcamp
  • Best Online Coding Bootcamps and Courses
  • Best Free Bootcamps and Coding Training
  • Coding Bootcamp vs. Community College
  • Coding Bootcamp vs. Self-Learning
  • Bootcamps vs. Certifications: Compared
  • What Is a Coding Bootcamp Job Guarantee?
  • How to Pay for Coding Bootcamp
  • Ultimate Guide to Coding Bootcamp Loans
  • Best Coding Bootcamp Scholarships and Grants
  • Education Stipends for Coding Bootcamps
  • Get Your Coding Bootcamp Sponsored by Your Employer
  • GI Bill and Coding Bootcamps
  • Tech Intevriews
  • Our Enterprise Solution
  • Connect With Us
  • Publication
  • Reskill America
  • Partner With Us

Career Karma

  • Resource Center
  • Bachelor’s Degree
  • Master’s Degree

Python indexerror: list assignment index out of range Solution

An IndexError is nothing to worry about. It’s an error that is raised when you try to access an index that is outside of the size of a list. How do you solve this issue? Where can it be raised?

In this article, we’re going to answer those questions. We will discuss what IndexErrors are and how you can solve the “list assignment index out of range” error. We’ll walk through an example to help you see exactly what causes this error.

Find your bootcamp match

Without further ado, let’s begin!

The Problem: indexerror: list assignment index out of range

When you receive an error message, the first thing you should do is read it. An error message can tell you a lot about the nature of an error.

Our error message is: indexerror: list assignment index out of range.

IndexError tells us that there is a problem with how we are accessing an index . An index is a value inside an iterable object, such as a list or a string.

The message “list assignment index out of range” tells us that we are trying to assign an item to an index that does not exist.

In order to use indexing on a list, you need to initialize the list. If you try to assign an item into a list at an index position that does not exist, this error will be raised.

An Example Scenario

The list assignment error is commonly raised in for and while loops .

We’re going to write a program that adds all the cakes containing the word “Strawberry” into a new array. Let’s start by declaring two variables:

The first variable stores our list of cakes. The second variable is an empty list that will store all of the strawberry cakes. Next, we’re going to write a loop that checks if each value in “cakes” contains the word “Strawberry”.

If a value contains “Strawberry”, it should be added to our new array. Otherwise, nothing will happen. Once our for loop has executed, the “strawberry” array should be printed to the console. Let’s run our code and see what happens:

As we expected, an error has been raised. Now we get to solve it!

The Solution

Our error message tells us the line of code at which our program fails:

The problem with this code is that we are trying to assign a value inside our “strawberry” list to a position that does not exist.

When we create our strawberry array, it has no values. This means that it has no index numbers. The following values do not exist:

We are trying to assign values to these positions in our for loop. Because these positions contain no values, an error is returned.

We can solve this problem in two ways.

Solution with append()

First, we can add an item to the “strawberry” array using append() :

The append() method adds an item to an array and creates an index position for that item. Let’s run our code: [‘Strawberry Tart’, ‘Strawberry Cheesecake’].

Our code works!

Solution with Initializing an Array

Alternatively, we can initialize our array with some values when we declare it. This will create the index positions at which we can store values inside our “strawberry” array.

To initialize an array, you can use this code:

This will create an array with 10 empty values. Our code now looks like this:

Let’s try to run our code:

Our code successfully returns an array with all the strawberry cakes.

This method is best to use when you know exactly how many values you’re going to store in an array.

Venus profile photo

"Career Karma entered my life when I needed it most and quickly helped me match with a bootcamp. Two months after graduating, I found my dream job that aligned with my values and goals in life!"

Venus, Software Engineer at Rockbot

Our above code is somewhat inefficient because we have initialized “strawberry” with 10 empty values. There are only a total of three cakes in our “cakes” array that could possibly contain “Strawberry”. In most cases, using the append() method is both more elegant and more efficient.

IndexErrors are raised when you try to use an item at an index value that does not exist. The “indexerror: list assignment index out of range” is raised when you try to assign an item to an index position that does not exist.

To solve this error, you can use append() to add an item to a list. You can also initialize a list before you start inserting values to avoid this error.

Now you’re ready to start solving the list assignment error like a professional Python developer!

About us: Career Karma is a platform designed to help job seekers find, research, and connect with job training programs to advance their careers. Learn about the CK publication .

What's Next?

icon_10

Get matched with top bootcamps

Ask a question to our community, take our careers quiz.

James Gallagher

Leave a Reply Cancel reply

Your email address will not be published. Required fields are marked *

Apply to top tech training programs in one click

FEATURES

  • Documentation
  • System Status

Resources

  • Rollbar Academy

Events

  • Software Development
  • Engineering Management
  • Platform/Ops
  • Customer Support
  • Software Agency

Use Cases

  • Low-Risk Release
  • Production Code Quality
  • DevOps Bridge
  • Effective Testing & QA

How to Fix “IndexError: List Assignment Index Out of Range” in Python

How to Fix “IndexError: List Assignment Index Out of Range” in Python

Table of Contents

The IndexError: List Assignment Index Out of Range error occurs when you assign a value to an index that is beyond the valid range of indices in the list. As Python uses zero-based indexing, when you try to access an element at an index less than 0 or greater than or equal to the list’s length, you trigger this error.

It’s not as complicated as it sounds. Think of it this way: you have a row of ten mailboxes, numbered from 0 to 9. These mailboxes represent the list in Python. Now, if you try to put a letter into mailbox number 10, which doesn't exist, you'll face a problem. Similarly, if you try to put a letter into any negative number mailbox, you'll face the same issue because those mailboxes don't exist either.

The IndexError: List Assignment Index Out of Range error in Python is like trying to put a letter into a mailbox that doesn't exist in our row of mailboxes. Just as you can't access a non-existent mailbox, you can't assign a value to an index in a list that doesn't exist.

Let’s take a look at example code that raises this error and some strategies to prevent it from occurring in the first place.

Example of “IndexError: List Assignment Index Out of Range”

Remember, assigning a value at an index that is negative or out of bounds of the valid range of indices of the list raises the error.

How to resolve “IndexError: List Assignment Index Out of Range”

You can use methods such as append() or insert() to insert a new element into the list.

How to use the append() method

Use the append() method to add elements to extend the list properly and avoid out-of-range assignments.

How to use the insert() method

Use the insert() method to insert elements at a specific position instead of direct assignment to avoid out-of-range assignments.

Now one big advantage of using insert() is even if you specify an index position which is way out of range it won’t give any error and it will just append the element at the end of the list.

Track, Analyze and Manage Errors With Rollbar

Managing errors and exceptions in your code is challenging. It can make deploying production code an unnerving experience. Being able to track, analyze, and manage errors in real-time can help you proceed with more confidence. Rollbar automates error monitoring and triaging, making fixing Python errors easier than ever. Try it today !

Related Resources

How to Handle TypeError: Cannot Unpack Non-iterable Nonetype Objects in Python

How to Handle TypeError: Cannot Unpack Non-iterable Nonetype Objects in Python

How to Fix IndexError: string index out of range in Python

How to Fix IndexError: string index out of range in Python

How to Fix Python’s “List Index Out of Range” Error in For Loops

How to Fix Python’s “List Index Out of Range” Error in For Loops

"Rollbar allows us to go from alerting to impact analysis and resolution in a matter of minutes. Without it we would be flying blind."

Error Monitoring

Start continuously improving your code today.

Datagy logo

  • Learn Python
  • Python Lists
  • Python Dictionaries
  • Python Strings
  • Python Functions
  • Learn Pandas & NumPy
  • Pandas Tutorials
  • Numpy Tutorials
  • Learn Data Visualization
  • Python Seaborn
  • Python Matplotlib

Python IndexError: List Index Out of Range Error Explained

  • November 15, 2021 December 19, 2022

Python IndexError Cover Image

In this tutorial, you’ll learn how all about the Python list index out of range error, including what it is, why it occurs, and how to resolve it.

The IndexError is one of the most common Python runtime errors that you’ll encounter in your programming journey. For the most part, these these errors are quite easy to resolve, once you understand why they occur.

Throughout this tutorial, you’ll learn why the error occurs and walk through some scenarios where you might encounter it. You’ll also learn how to resolve the error in these scenarios .

The Quick Answer:

Table of Contents

What is the Python IndexError?

Let’s take a little bit of time to explore what the Python IndexError is and what it looks like. When you encounter the error, you’ll see an error message displayed as below:

We can break down the text a little bit. We can see here that the message tells us that the index is out of range . This means that we are trying to access an index item in a Python list that is out of range, meaning that an item doesn’t have an index position.

An item that doesn’t have an index position in a Python list, well, doesn’t exist.

In Python, like many other programming languages, a list index begins at position 0 and continues to n-1 , where n is the length of the list (or the number of items in that list).

This causes a fairly common error to occur. Say we are working with a list with 4 items. If we wanted to access the fourth item, you may try to do this by using the index of 4. This, however, would throw the error. This is because the 4 th item actually has the index of 3.

Let’s take a look at a sample list and try to access an item that doesn’t exist:

We can see here that the index error occurs on the last item we try to access.

The simplest solution is to simply not try to access an item that doesn’t exist . But that’s easier said than done. How do we prevent the IndexError from occurring? In the next two sections, you’ll learn how to fix the error from occurring in their most common situations: Python for loops and Python while loops.

Need to check if a key exists in a Python dictionary? Check out this tutorial , which teaches you five different ways of seeing if a key exists in a Python dictionary, including how to return a default value.

Python IndexError with For Loop

You may encounter the Python IndexError while running a Python for loop. This is particularly common when you try to loop over the list using the range() function .

Let’s take a look at the situation where this error would occur:

The way that we can fix this error from occurring is to simply stop the iteration from occurring before the list runs out of items . The way that we can do this is to change our for loop from going to our length + 1, to the list’s length. When we do this, we stop iterating over the list’s indices before the lengths value.

This solves the IndexError since it causes the list to stop iterating at position length - 1 , since our index begins at 0, rather than at 1.

Let’s see how we can change the code to run correctly:

Now that you have an understanding of how to resolve the Python IndexError in a for loop, let’s see how we can resolve the error in a Python while-loop.

Want to learn more about Python for-loops? Check out my in-depth tutorial that takes your from beginner to advanced for-loops user! Want to watch a video instead? Check out my YouTube tutorial here .

Python IndexError with While Loop

You may also encounter the Python IndexError when running a while loop.

For example, it may be tempting to run a while loop to iterate over each index position in a list. You may, for example, write a program that looks like this:

The reason that this program fails is that we iterate over the list one too many times. The reason this is true is that we are using a <= (greater than or equal to sign). Because Python list indices begin at the value 0, their max index is actually equal to the number of items in the list minus 1.

We can resolve this by simply changing the operator a less than symbol, < . This prevents the loop from looping over the index from going out of range.

In the next section, you'll learn a better way to iterate over a Python list to prevent the IndexError .

Want to learn more about Python f-strings? Check out my in-depth tutorial , which includes a step-by-step video to master Python f-strings!

How to Fix the Python IndexError

There are two simple ways in which you can iterate over a Python list to prevent the Python IndexError .

The first is actually a very plain language way of looping over a list. We don't actually need the list index to iterate over a list. We can simply access its items directly.

This directly prevents Python from going beyond the maximum index.

Want to learn how to use the Python zip() function to iterate over two lists? This tutorial teaches you exactly what the zip() function does and shows you some creative ways to use the function.

But what if you need to access the list's index?

If you need to access the list's index and a list item, then a much safer alternative is to use the Python enumerate() function.

When you pass a list into the enumerate() function, an enumerate object is returned. This allows you to access both the index and the item for each item in a list. The function implicitly stops at the maximum index, but allows you to get quite a bit of information.

Let's take a look at how we can use the enumerate() function to prevent the Python IndexError .

We can see here that we the loop stops before the index goes out of range and thereby prevents the Python IndexError .

Check out some other Python tutorials on datagy, including our complete guide to styling Pandas and our comprehensive overview of Pivot Tables in Pandas !

In this tutorial, you learned how to understand the Python IndexError : list item out of range. You learned why the error occurs, including some common scenarios such as for loops and while loops. You learned some better ways of iterating over a Python list, such as by iterating over items implicitly as well as using the Python enumerate() function.

To learn more about the Python IndexError , check out the official documentation here .

Nik Piepenbreier

Nik is the author of datagy.io and has over a decade of experience working with data analytics, data science, and Python. He specializes in teaching developers how to use Python for data science using hands-on tutorials. View Author posts

1 thought on “Python IndexError: List Index Out of Range Error Explained”

' src=

from django.contrib import messages from django.shortcuts import render, redirect

from home.forms import RewardModeLForm from item.models import Item from person.models import Person from .models import Reward, YoutubeVideo # Create your views here.

def home(request): my_reward = Reward.objects.all()[:1] # First Div last_person_post = Person.objects.all()[:1] last_item_post = Item.objects.all()[:1] # 2nd Div lost_person = Person.objects.filter(person=”L”).all()[:1] lost_item = Item.objects.filter(category=”L”).all()[:2] # End 2 div

home_found = Person.objects.all()[:3] home_item = Item.objects.all()[:3] videos = YoutubeVideo.objects.all()[:3] context = { ‘my_reward’: my_reward, ‘lost_person’: lost_person, ‘lost_item’: lost_item, ‘home_found’: home_found, ‘home_item’: home_item, ‘videos’: videos, } if last_person_post[0].update > last_item_post[0].update: context[‘last_post’] = last_person_post else: context[‘last_post’] = last_item_post

return render(request, ‘home/home.html’, context)

# Reward Function

def reward(request): if request.method == ‘POST’: form = RewardModeLForm(request.POST or None) if form.is_valid(): instance = form.save(commit=False) instance.user = request.user instance.save() messages.add_message(request, messages.SUCCESS, ‘Reward Updated .’) return redirect(‘home’) else: form = RewardModeLForm() context = { ‘form’: form, } return render(request, ‘home/reward.html’, context) index out of rage

Leave a Reply Cancel reply

Your email address will not be published. Required fields are marked *

Save my name, email, and website in this browser for the next time I comment.

  • Python Basics
  • Interview Questions
  • Python Quiz
  • Popular Packages
  • Python Projects
  • Practice Python
  • AI With Python
  • Learn Python3
  • Python Automation
  • Python Web Dev
  • DSA with Python
  • Python OOPs
  • Dictionaries
  • Python List Index Out of Range - How to Fix IndexError
  • Python Indexerror: list assignment index out of range Solution
  • How to Fix – Indexerror: Single Positional Indexer Is Out-Of-Bounds
  • IndexError: pop from Empty List in Python
  • Creating a list of range of dates in Python
  • How to fix "'list' object is not callable" in Python
  • Python | Assign range of elements to List
  • Split a Python List into Sub-Lists Based on Index Ranges
  • How to Access Index in Python's for Loop
  • How to iterate through a nested List in Python?
  • How to Find the Index for a Given Item in a Python List
  • range() to a list in Python
  • How to Replace Values in a List in Python?
  • Python - Product of elements using Index list
  • Internal working of list in Python
  • Python | Ways to find indices of value in list
  • How we can iterate through list of tuples in Python
  • Python Program to get indices of sign change in a list
  • How to Remove an Item from the List in Python
  • Python | Numbers in a list within a given range

Python List Index Out of Range – How to Fix IndexError

In Python, the IndexError is a common exception that occurs when trying to access an element in a list, tuple, or any other sequence using an index that is outside the valid range of indices for that sequence. List Index Out of Range Occur in Python when an item from a list is tried to be accessed that is outside the range of the list. Before we proceed to fix the error, let’s discuss how indexing work in Python .

What Causes an IndexError in Python

  • Accessing Non-Existent Index: When you attempt to access an index of a sequence (such as a list or a string) that is out of range, an Indexerror is raised. Sequences in Python are zero-indexed, which means that the first element’s index is 0, the second element’s index is 1, and so on.
  • Empty List: If you try to access an element from an empty list, an Indexerror will be raised since there are no elements in the list to access.

Example: Here our list is 3 and we are printing with size 4 so in this case, it will create a list index out of range.

Similarly, we can also get an Indexerror when using negative indices.

How to Fix IndexError in Python

  • Check List Length: It’s important to check if an index is within the valid range of a list before accessing an element. To do so, you can use the function to determine the length of the list and make sure the index falls within the range of 0 to length-1.
  • Use Conditional Statements: To handle potential errors, conditional statements like “if” or “else” blocks can be used. For example, an “if” statement can be used to verify if the index is valid before accessing the element. if or try-except blocks to handle the potential IndexError . For instance, you can use a if statement to check if the index is valid before accessing the element.

How to Fix List Index Out of Range in Python

Let’s see some examples that showed how we may solve the error.

  • Using Python range()
  • Using Python Index()
  • Using Try Except Block

Python Fix List Index Out of Range using Range()

The range is used to give a specific range, and the Python range() function returns the sequence of the given number between the given range.

Python Fix List Index Out of Range u sing Index()

Here we are going to create a list and then try to iterate the list using the constant values in for loops.

Reason for the error –  The length of the list is 5 and if we are an iterating list on 6 then it will generate the error.

Solving this error without using Python len() or constant Value: To solve this error we will take the index of the last value of the list and then add one then it will become the exact value of length.

Python Fix List Index Out of Range using Try Except Block

If we expect that an index might be out of range, we can use a try-except block to handle the error gracefully.

Please Login to comment...

Similar reads.

  • Python How-to-fix
  • python-list

Improve your Coding Skills with Practice

 alt=

What kind of Experience do you want to share?

List Index Out of Range – Python Error Message Solved

Dionysia Lemonaki

In this article you'll see a few of the reasons that cause the list index out of range Python error.

Besides knowing why this error occurs in the first place, you'll also learn some ways to avoid it.

Let's get started!

How to Create a List in Python

To create a list object in Python, you need to:

  • Give the list a name,
  • Use the assignment operator, = ,
  • and include 0 or more list items inside square brackets, [] . Each list item needs to be separated by a comma.

For example, to create a list of names you would do the following:

The code above created a list called names that has four values: Kelly, Nelly, Jimmy, Lenny .

How to Check the Length of a List in Python

To check the length of a list in Python, use Python's build-in len() method.

len() will return an integer, which will be the number of items stored in the list.

There are four items stored in the list, therefore the length of the list will be four.

How to Access Individual List Items in Python

Each item in a list has its own index number .

Indexing in Python, and most modern programming languages, starts at 0.

This means that the first item in a list has an index of 0, the second item has an index of 1, and so on.

You can use the index number to access the individual item.

To access an item in a list using its index number, first write the name of the list. Then, inside square brackets, include the intiger that corresponds with the item's index number.

Taking the example from earlier, this is how you would access each item inside the list using its index number:

You can also use negative indexing to access items inside lists in Python.

To access the last item, you use the index value of -1. To acces the second to last item, you use the index value of -2.

Here is how you would access each item inside a list using negative indexing:

Why does the Indexerror: list index out of range error occur in Python?

Using an index number that is out of the range of the list.

You'll get the Indexerror: list index out of range error when you try and access an item using a value that is out of the index range of the list and does not exist.

This is quite common when you try to access the last item of a list, or the first one if you're using negative indexing.

Let's go back to the list we've used so far.

Say I want to access the last item, "Lenny", and try to do so by using the following code:

Generally, the index range of a list is 0 to n-1 , with n being the total number of values in the list.

With the total values of the list above being 4 , the index range is 0 to 3 .

Now, let's try to access an item using negative indexing.

Say I want to access the first item in the list, "Kelly", by using negative indexing.

When using negative indexing, the index range of a list is -1 to -n , where -n the total number of items contained in the list.

With the total number of items in the list being 4 , the index range is -1 to -4 .

Using the wrong value in the range() function in a Python for loop

You'll get the Indexerror: list index out of range error when iterating through a list and trying to access an item that doesn't exist.

One common instance where this can occur is when you use the wrong integer in Python's range() function.

The range() function typically takes in one integer number, which indicates where the counting will stop.

For example, range(5) indicates that the counting will start from 0 and end at 4 .

So, by default, the counting starts at position 0 , is incremented by 1 each time, and the number is up to – but not including – the position where the counting will stop.

Let's take the following example:

Here, the list names has four values.

I wanted to loop through the list and print out each value.

When I used range(5) I was telling the Python interpreter to print the values that are at the positions 0 to 4 .

However, there is no item in position 4.

You can see this by first printing out the number of the position and then the value at that position.

You see that at position 0 is "Kelly", at position 1 is "Nelly", at position 2 is "Jimmy" and at position 3 is "Lenny".

When it comes to position four, which was specified with range(5) which indicates positions of 0 to 4 , there is nothing to print out and therefore the interpreter throws an error.

One way to fix this is to lower the integer in range() :

Another way to fix this when using a for loop is to pass the length of the list as an argument to the range() function. You do this by using the len() built-in Python function, as shown in an earlier section:

When passing len() as an argument to range() , make sure that you don't make the following mistake:

After running the code, you'll again get an IndexError: list index out of range error:

Hopefully this article gave you some insight into why the IndexError: list index out of range error occurs and some ways you can avoid it.

If you want to learn more about Python, check out freeCodeCamp's Python Certification . You'll start learning in an interacitve and beginner-friendly way. You'll also build five projects at the end to put into practice and help reinforce what you learned.

Thanks for reading and happy coding!

Read more posts .

If this article was helpful, share it .

Learn to code for free. freeCodeCamp's open source curriculum has helped more than 40,000 people get jobs as developers. Get started

How to Fix Python IndexError: list assignment index out of range

  • Python How-To's
  • How to Fix Python IndexError: list …

Python IndexError: list assignment index out of range

Fix the indexerror: list assignment index out of range in python, fix indexerror: list assignment index out of range using append() function, fix indexerror: list assignment index out of range using insert() function.

How to Fix Python IndexError: list assignment index out of range

In Python, the IndexError: list assignment index out of range is raised when you try to access an index of a list that doesn’t even exist. An index is the location of values inside an iterable such as a string, list, or array.

In this article, we’ll learn how to fix the Index Error list assignment index out-of-range error in Python.

Let’s see an example of the error to understand and solve it.

Code Example:

The reason behind the IndexError: list assignment index out of range in the above code is that we’re trying to access the value at the index 3 , which is not available in list j .

To fix this error, we need to adjust the indexing of iterables in this case list. Let’s say we have two lists, and you want to replace list a with list b .

You cannot assign values to list b because the length of it is 0 , and you are trying to add values at kth index b[k] = I , so it is raising the Index Error. You can fix it using the append() and insert() .

The append() function adds items (values, strings, objects, etc.) at the end of the list. It is helpful because you don’t have to manage the index headache.

The insert() function can directly insert values to the k'th position in the list. It takes two arguments, insert(index, value) .

In addition to the above two solutions, if you want to treat Python lists like normal arrays in other languages, you can pre-defined your list size with None values.

Once you have defined your list with dummy values None , you can use it accordingly.

There could be a few more manual techniques and logic to handle the IndexError: list assignment index out of range in Python. This article overviews the two common list functions that help us handle the Index Error in Python while replacing two lists.

We have also discussed an alternative solution to pre-defined the list and treat it as an array similar to the arrays of other programming languages.

Zeeshan Afridi avatar

Zeeshan is a detail oriented software engineer that helps companies and individuals make their lives and easier with software solutions.

Related Article - Python Error

  • Can Only Concatenate List (Not Int) to List in Python
  • How to Fix Value Error Need More Than One Value to Unpack in Python
  • How to Fix ValueError Arrays Must All Be the Same Length in Python
  • Invalid Syntax in Python
  • How to Fix the TypeError: Object of Type 'Int64' Is Not JSON Serializable
  • How to Fix the TypeError: 'float' Object Cannot Be Interpreted as an Integer in Python

Related Article - Python List

  • How to Convert a Dictionary to a List in Python
  • How to Remove All the Occurrences of an Element From a List in Python
  • How to Remove Duplicates From List in Python
  • How to Get the Average of a List in Python
  • What Is the Difference Between List Methods Append and Extend
  • How to Convert a List to String in Python
  • Data Analysis
  • Deep Learning
  • Large Language Model
  • Machine Learning
  • Neural Networks

Logo

Resolve "index out of range" errors

As a Python developer, working with lists is an essential part of your daily coding routine. However, even experienced programmers can stumble upon the dreaded “index out of range” error when dealing with list assignments. This error occurs when you attempt to access or modify an index that doesn’t exist within the list’s bounds. Fear not, as this comprehensive tutorial will equip you with the knowledge and techniques to conquer this challenge and unlock the full potential of Python’s list assignment.

Understanding the “Index Out of Range” Error

Before diving into the solutions, let’s first understand the root cause of the “index out of range” error. In Python, lists are zero-indexed, meaning the first element has an index of 0, the second element has an index of 1, and so on. When you try to access or modify an index that falls outside the list’s valid range, Python raises an IndexError with the “index out of range” message.

Here’s an example that illustrates the error:

In this case, we’re attempting to access the fourth element ( my_list[3] ) of a list that only contains three elements (indices 0, 1, and 2).

Solution 1: Validating Indices Before Assignment

One effective solution to prevent “index out of range” errors is to validate the index before attempting to assign a value to it. You can achieve this by checking if the index falls within the list’s valid range using the len() function and conditional statements.

In this solution, we first declare a list my_list with three elements. We then define two variables: index and new_value .

Next, we use an if statement to check if the index is less than the length of my_list . The len(my_list) function returns the number of elements in the list, which is 3 in this case.

If the condition index < len(my_list) is True, it means that the index is a valid index within the list’s bounds. In this case, we assign the new_value (4) to the element at the specified index (2) using the list assignment my_list[index] = new_value . Finally, we print the updated list, which now has the value 4 at index 2.

However, if the condition index < len(my_list) is False, it means that the index is out of range for the given list. In this case, we execute the else block and print the message “Index out of range!” to inform the user that the provided index is invalid.

This solution is effective when you need to ensure that the index you’re trying to access or modify is within the valid range of the list. By checking the index against the list’s length , you can prevent “index out of range” errors and handle invalid indices appropriately.

It’s important to note that this solution assumes that the index variable is provided or calculated elsewhere in the code. In a real-world scenario, you may need to handle user input or perform additional validation on the index variable to ensure it’s an integer and within the expected range.

Solution 2: Using Python’s List Slicing

Python’s list slicing feature allows you to access and modify a subset of elements within a list. This is a powerful technique that can help you avoid “index out of range” errors when working with list assignments .

In the first example, my_list[:2] = [10, 20] , we’re using list slicing to access and modify the elements from the start of the list up to (but not including) index 2. The slice [:2] represents the range from the beginning of the list to index 2 (0 and 1). We then assign the new values [10, 20] to this slice, effectively replacing the original values at indices 0 and 1 with 10 and 20, respectively.

In the second example, my_list[2:] = [30, 40] , we’re using list slicing to access and modify the elements from index 2 to the end of the list. The slice [2:] represents the range from index 2 to the end of the list. We then assign the new values [30, 40] to this slice. Since the original list only had three elements, Python automatically extends the list by adding a new element at index 3 to accommodate the second value (40).

List slicing is a powerful technique because it allows you to modify multiple elements within a list without worrying about “index out of range” errors. Python automatically handles the indices for you, ensuring that the assignment operation is performed within the valid range of the list.

Here are a few key points about list slicing:

  • Inclusive start, exclusive end : The slice [start:end] includes the elements from start up to, but not including, end .
  • Omitting start or end : If you omit the start index, Python assumes the beginning of the list. If you omit the end index, Python assumes the end of the list.
  • Negative indices : You can use negative indices to start or end the slice from the end of the list. For example, my_list[-1] accesses the last element of the list.
  • Step size : You can optionally specify a step size in the slice notation, e.g., my_list[::2] to access every other element of the list.

List slicing is a powerful and Pythonic way to work with lists, and it can help you avoid “index out of range” errors when assigning values to multiple elements within a list.

Solution 3: Using Python’s List Append Method

The append() method in Python is a built-in list method that allows you to add a new element to the end of an existing list. This method is particularly useful when you want to avoid “index out of range” errors that can occur when trying to assign a value to an index that doesn’t exist in the list.

In this example, we start with a list my_list containing three elements: [1, 2, 3] . We then use the append() method to add a new element 4 to the end of the list: my_list.append(4) . Finally, we print the updated list, which now contains four elements: [1, 2, 3, 4] .

Here’s how the append() method works:

  • Python finds the current length of the list using len(my_list) .
  • It assigns the new value ( 4 in this case) to the index len(my_list) , which is the next available index after the last element in the list.
  • Since the new index is always valid (it’s one greater than the last index), there’s no risk of an “index out of range” error.

The append() method is a safe and convenient way to add new elements to the end of a list because it automatically handles the index assignment for you. You don’t need to worry about calculating the correct index or checking if the index is within the list’s bounds.

It’s important to note that the append() method modifies the original list in-place. If you want to create a new list instead of modifying the existing one, you can use the + operator or the list.copy() method to create a copy of the list first, and then append the new element to the copy.

Another advantage of using append() is that it allows you to add multiple elements to the list in a loop or by iterating over another sequence. For example:

In this example, we use a for loop to iterate over the new_elements list, and for each element, we call my_list.append(element) to add it to the end of my_list .

Overall, the append() method is a simple and effective way to add new elements to the end of a list, ensuring that you avoid “index out of range” errors while maintaining the integrity and order of your list.

Solution 4: Handling Exceptions with Try/Except Blocks

Python provides a robust exception handling mechanism using try/except blocks, which can be used to gracefully handle “index out of range” errors and other exceptions that may occur during program execution.

In this example, we first define a list my_list with three elements and an index variable with the value 4 .

The try block contains the code that might raise an exception. In this case, we attempt to access the element at my_list[index] , which is my_list[4] . Since the list only has indices from 0 to 2, this operation will raise an IndexError with the message “list index out of range” .

The except block specifies the type of exception to catch, which is IndexError in this case. If an IndexError is raised within the try block, the code inside the except block will be executed. Here, we print the message “Index out of range! Please provide a valid index.” to inform the user that the provided index is invalid.

If no exception is raised within the try block, the except block is skipped, and the program continues executing the code after the try/except block.

By using try/except blocks, you can handle exceptions gracefully and provide appropriate error messages or take alternative actions, rather than allowing the program to crash with an unhandled exception.

Here are a few key points about using try/except blocks for handling exceptions:

  • Multiple except blocks : You can have multiple except blocks to handle different types of exceptions. This allows you to provide specific error handling for each exception type.
  • Exception objects : The except block can optionally include a variable to hold the exception object, which can provide additional information about the exception.
  • else clause : You can include an else clause after the except blocks. The else block executes if no exceptions are raised in the try block.
  • finally clause : The finally clause is executed regardless of whether an exception was raised or not. It’s typically used for cleanup operations, such as closing files or releasing resources.
  • Exception hierarchy : Python has a built-in exception hierarchy, where some exceptions are derived from others. You can catch a base exception to handle multiple related exceptions or catch specific exceptions for more granular control.

By using try/except blocks and handling exceptions properly, you can write more robust and resilient code that gracefully handles errors, making it easier to debug and maintain your Python programs.

Best Practices and Coding Standards

To ensure your code is not only functional but also maintainable and scalable, it’s essential to follow best practices and coding standards. Here are some recommendations:

  • Validate user input : When working with user-provided indices, always validate the input to ensure it falls within the list’s valid range.
  • Use descriptive variable and function names : Choose meaningful names that clearly convey the purpose and functionality of your code elements.
  • Write clear and concise comments : Document your code with comments that explain the purpose, logic, and any non-obvious implementation details.
  • Follow PEP 8 style guide : Adhere to Python’s official style guide , PEP 8, to ensure consistency and readability across your codebase.
  • Test your code thoroughly : Implement unit tests and integrate testing into your development workflow to catch bugs and regressions early.

By following these best practices and coding standards, you’ll not only avoid “index out of range” errors but also produce high-quality, maintainable, and scalable Python code.

Mastering Python’s list assignment is crucial for efficient data manipulation and programming success. By understanding the root cause of “index out of range” errors and implementing the solutions outlined in this tutorial, you’ll be well-equipped to handle these challenges confidently. Whether you validate indices, leverage list slicing, use the append() method, or handle exceptions, you now have a comprehensive toolkit to tackle list assignment challenges head-on. Embrace these techniques, follow best practices, and continue honing your Python skills to unlock new levels of coding excellence.

Related Posts

  • Hierarchical Cluster Analysis: How it is Used for Data Analysis
  • Data Backup & Recovery: How to Use Data Analysis to Protect Data from Loss in Case of Failure or Disaster
  • Install WordPress with Docker Compose – Easy Guide
  • Python zlib: Compress and Decompress Data Efficiently
  • Install Htop Viewer on Ubuntu 22.04 or 20.04
  • Index Out of Range
  • List Assignment

Python Image Processing With OpenCV

Install python 3.10 on centos/rhel 8 & fedora 35/34, how to overwrite a file in python, why is python so popular, itertools combinations – python, colorama in python, matplotlib log scale in python, how to generate dummy data with python faker, more article, python file stat() simplified: a comprehensive guide to file metadata, boost your website rankings with python and ai seo techniques, python atan() and atan2() functions, mastering the python math.atan2() method.

2016 began to contact WordPress, the purchase of Web hosting to the installation, nothing, step by step learning, the number of visitors to the site, in order to save money, began to learn VPS. Linux, Ubuntu, Centos …

Popular Posts

Master wifi control on linux with networkmanager cli wizardry, install and use onenote on ubuntu 24.04, popular categories.

  • Artificial Intelligence 320
  • Data Analysis 205
  • Security 91
  • Privacy Policy
  • Terms & Conditions

©markaicode.com. All rights reserved - 2022 by Mark

解决:python中出现:list assignment index out of range

list assignment is out of range

Traceback (most recent call last):   File "E:/testPython/NumPY/test.py", line 67, in <module>     readIp()   File "E:/testPython/NumPY/test.py", line 59, in readIp     iplist[i]=ip IndexError: list assignment index out of range

原始代码如下:

list assignment index out of range:列表超过限制

一种情况是:list[index]index超出范围

另一种情况是:list是一个空的,没有一个元素,进行list[0]就会出现错误!

解决办法如下:

将iplist=[]   改为:iplist={}

list assignment is out of range

“相关推荐”对你有帮助么?

list assignment is out of range

请填写红包祝福语或标题

list assignment is out of range

1.余额是钱包充值的虚拟货币,按照1:1的比例进行支付金额的抵扣。 2.余额无法直接购买下载,可以购买VIP、付费专栏及课程。

list assignment is out of range

Consultancy

  • Technology Consulting
  • Customer Experience Consulting
  • Solution Architect Consulting

Software Development Services

  • Ecommerce Development
  • Web App Development
  • Mobile App Development
  • SAAS Product Development
  • Content Management System
  • System Integration & Data Migration
  • Cloud Computing
  • Computer Vision

Dedicated Development Team

  • Full Stack Developers For Hire
  • Offshore Development Center

Marketing & Creative Design

  • UX/UI Design
  • Customer Experience Optimization
  • Digital Marketing
  • Devops Services
  • Service Level Management
  • Security Services
  • Odoo gold partner

By Industry

  • Retail & Ecommerce
  • Manufacturing
  • Import & Distribution
  • Financical & Banking
  • Technology For Startups

Business Model

  • MARKETPLACE ECOMMERCE

Our realized projects

list assignment is out of range

MB Securities - A Premier Brokerage

list assignment is out of range

iONAH - A Pioneer in Consumer Electronics Industry

list assignment is out of range

Emers Group - An Official Nike Distributing Agent

list assignment is out of range

Academy Xi - An Australian-based EdTech Startup

  • Market insight

list assignment is out of range

  • Ohio Digital
  • Onnet Consoulting

></center></p><h2>List assignment index out of range: Python indexerror solution you should know</h2><p>An IndexError is nothing to worry about. In this article, we’re going to give you the Python indexerror solution to list assignment index out of range. We will also walk through an example to help you see exactly what causes this error. Souce: careerkarma</p><p><center><img style=

The Problem: indexerror: list assignment index out of range

When you receive an error message, the first thing you should do is read it. Because, an error message can tell you a lot about the nature of an error.

indexer message is: 

indexerror: list assignment index out of range.

To clarify, IndexError tells us that there is a problem with how we are accessing an index. An index is a value inside an iterable object, such as a list or a string. Then, the message “list assignment index out of range” tells us that we are trying to assign an item to an index that does not exist.

In order to use indexing on a list, you need to initialize the list. Moreover, if you try to assign an item into a list at an index position that does not exist, this error will be raised.

An Example Scenario

The list assignment error is commonly raised in for and while loops.

We’re going to write a program that adds all the cakes containing the word “Strawberry” into a new array. Let’s start by declaring two variables:

To clarify, the first variable stores our list of cakes. The second variable is an empty list that will store all of the strawberry cakes. Then, we’re going to write a loop that checks if each value in “cakes” contains the word “Strawberry”.

If a value contains “Strawberry”, it should be added to our new array. Otherwise, nothing will happen. Once our for loop has executed, the “strawberry” array should be printed to the console. Let’s run our code and see what happens:

As we expected, an error has been raised. Then, we get to solve it.

>>> Read more

  • Local variable referenced before assignment: The UnboundLocalError in Python
  • Rename files using Python: How to implement it with examples

The solution to list assignment Python index out of range

Our error message tells us the line of code at which our program fails:

To clarify, the problem with this code is that we are trying to assign a value inside our “strawberry” list to a position that does not exist.

When we create our strawberry array, it has no values. To clarify, this means that it has no index numbers. The following values do not exist:

We are trying to assign values to these positions in our for loop. Because these positions contain no values, an error is returned. So, we can solve this problem in two ways.

Solution with append()

Firstly, we can add an item to the “strawberry” array using append():

The  append()  method adds an item to an array and creates an index position for that item.

Let’s run our code:

The code works!

Solution with Initializing an Array to list assignment Python index out of range

Alternatively, we can initialize our array with some values when we declare it. Because, Tthis will create the index positions at which we can store values inside our “strawberry” array. Therefore, to initialize an array, you can use this code:

This will create an array with 10 empty values. Our code now looks like this:

Let’s try to run the code:

The code successfully returns an array with all the strawberry cakes.

This method is best to use when you know exactly how many values you’re going to store in an array.

The above code is somewhat inefficient because we have initialized “strawberry” with 10 empty values. There are only a total of three cakes in our “cakes” array that could possibly contain “Strawberry”.

To sum up with list assignment python index out of range

IndexErrors are raised when you try to use an item at an index value that does not exist. The “indexerror: list assignment index out of range” is raised when you try to assign an item to an index position that does not exist.

To solve this error, you can use  append()  to add an item to a list. You can also initialize a list before you start inserting values to avoid this error. So, now you’re ready to start solving the list assignment error like a professional Python developer .

Do you have trouble with contacting a developer? So we suggest you one of the leading IT Companies in Vietnam – AHT Tech . AHT Tech is the favorite pick of many individuals and corporations in the world. For that reason, let’s explore what awesome services which AHT Tech have? More importantly, don’t forget to CONTACT US if you need help with our services .

  • code review process , ecommerce web/app development , eCommerce web/mobile app development , fix error , fix python error , list assignment index out of range , python indexerror , web/mobile app development

Our Other Services

  • E-commerce Development
  • Web Apps Development
  • Web CMS Development
  • Mobile Apps Development
  • Software Consultant & Development
  • System Integration & Data Migration
  • Dedicated Developers & Testers For Hire
  • Remote Working Team
  • Saas Products Development
  • Web/Mobile App Development
  • Outsourcing
  • Hiring Developers
  • Digital Transformation
  • Advanced SEO Tips

Offshore Development center

Lastest News

aws-cloud-managed-services

Challenges and Advices When Using AWS Cloud Managed Services

aws-well-architected-framework

Comprehensive Guide about AWS Well Architected Framework

aws-cloud-migration

AWS Cloud Migration: Guide-to-Guide from A to Z

cloud computing for healthcare

Uncover The Treasures Of Cloud Computing For Healthcare 

cloud computing in financial services

A Synopsis of Cloud Computing in Financial Services 

applications of cloud computing

Discover Cutting-Edge Cloud Computing Applications To Optimize Business Resources

Tailor your experience

  • Success Stories

Copyright ©2007 – 2021 by AHT TECH JSC. All Rights Reserved.

list assignment is out of range

Thank you for your message. It has been sent.

Navigation Menu

Search code, repositories, users, issues, pull requests..., provide feedback.

We read every piece of feedback, and take your input very seriously.

Saved searches

Use saved searches to filter your results more quickly.

To see all available qualifiers, see our documentation .

  • Notifications You must be signed in to change notification settings

Have a question about this project? Sign up for a free GitHub account to open an issue and contact its maintainers and the community.

By clicking “Sign up for GitHub”, you agree to our terms of service and privacy statement . We’ll occasionally send you account related emails.

Already on GitHub? Sign in to your account

IndexError: list assignment index out of range #134

@ramonyaskal

ramonyaskal commented Sep 26, 2022

self.validators[0] = MaxValueMultiFieldValidator(self.max_length)
IndexError: list assignment index out of range

django==4.1.1
django-multiselectfield==0.1.12

  • 👍 6 reactions

@mortenthansen

mortenthansen commented Oct 12, 2022

I see the same error after upgrading from django 4.0.7 to 4.1.2. This is a show-stopper.

Sorry, something went wrong.

@wbwlkr

wbwlkr commented Nov 17, 2022

Hello everyone,

: Same here 👋🏼
I fixed the issue by adding a to my .

Hopes this will help you all.

  • 👍 22 reactions

@imanshafiei540

imanshafiei540 commented May 27, 2023 • edited

I have the same issue here.

Python=3.8.16
Django==4.2.1
django-multiselectfield==0.1.12

And, probably, setting will solve my case too.

Happy to work on this issue if that's ok with the maintainers.

  • 👍 1 reaction

@pe712

pe712 commented Jul 21, 2023

Indeed we have :

(Field): description = _("String (up to %(max_length)s)") def __init__(self, *args, db_collation=None, **kwargs): super().__init__(*args, **kwargs) self.db_collation = db_collation if self.max_length is not None: self.validators.append(validators.MaxLengthValidator(self.max_length)) MultiSelectField(models.CharField): """ Choice values can not contain commas. """ def __init__(self, *args, **kwargs): self.min_choices = kwargs.pop('min_choices', None) self.max_choices = kwargs.pop('max_choices', None) super(MultiSelectField, self).__init__(*args, **kwargs) self.max_length = get_max_length(self.choices, self.max_length) self.validators[0] = MaxValueMultiFieldValidator(self.max_length)

We see that init passes to init. The is only appended to self.validators if not None. This causes the error.

In all cases, is later set to so the value we pass is deleted.

Moreover, , is the length of the in the database. And in the database the field is stored as for example. So is the length of all the fields concatenated. Which is exactly what returns.

To sum up, we should not need to specify in most cases. The package should be modified so that if is None, it is set to get_max_length AND the is appended.

@pe712

No branches or pull requests

@mortenthansen

What Part B covers

If you're in a Medicare Advantage Plan or other Medicare plan, your plan may have different rules. But, your plan must give you at least the same coverage as Original Medicare. Some services may only be covered in certain facilities or for patients with certain conditions.

What's covered?

NEW INSULIN BENEFIT!  If you use an insulin pump that's covered under Part B's durable medical equipment benefit, or you get your covered insulin through a Medicare Advantage Plan, your cost for a month's supply of Part B-covered insulin for your pump can't be more than $35. The Part B deductible won't apply. If you get a 3-month supply of Part B-covered insulin, your costs can't be more than $35 for each month's supply. This means you'll generally pay no more than $105 for a 3-month supply of covered insulin. If you have Part B and Medicare Supplement Insurance ( Medigap ) that pays your Part B coinsurance, you plan should cover the $35 (or less) cost for insulin.

Part B covers 2 types of services

  • Medically necessary services: Services or supplies that are needed to diagnose or treat your medical condition and that meet accepted standards of medical practice.
  • Preventive services :  Health care to prevent illness (like the flu) or detect it at an early stage, when treatment is most likely to work best.

You pay nothing for most preventive services if you get the services from a health care provider who accepts assignment .

Part B covers things like:

  • Clinical research  
  • Ambulance services
  • Durable medical equipment (DME)
  • Partial hospitalization
  • Intensive outpatient program services (starting January 1, 2024)
  • Limited outpatient prescription drugs

2 ways to find out if Medicare covers what you need

  • Talk to your doctor or other health care provider about why you need certain services or supplies. Ask if Medicare will cover them. You may need something that's usually covered but your provider thinks that Medicare won't cover it in your situation. If so, you'll have to  read and sign a notice . The notice says that you may have to pay for the item, service, or supply.
  • Find out if Medicare covers your item, service, or supply .

Medicare coverage is based on 3 main factors 

  • Federal and state laws.
  • National coverage decisions made by Medicare about whether something is covered.
  • Local coverage decisions made by companies in each state that process claims for Medicare. These companies decide whether something is medically necessary and should be covered in their area.

Advertisement

Supported by

Trump Has Few Ways to Overturn His Conviction as a New York Felon

The judge in Donald J. Trump’s case closed off many avenues of appeal, experts said, though his lawyers might challenge the novel theory at the case’s center.

  • Share full article

Donald Trump, on the sidewalk outside Trump Tower, points skyward.

By Ben Protess ,  William K. Rashbaum and Jonah E. Bromwich

“This is long from over,” Donald J. Trump, the former president and current felon, declared on Thursday, moments after a Manhattan jury convicted him on 34 counts of falsifying records to cover up a sex scandal.

Mr. Trump, the presumptive Republican nominee, is banking on the jury not having the final word on the case. He has already outlined a plan to appeal a verdict that on Friday he labeled “a scam.”

But even if the former — and possibly future — president could persuade voters to ignore his conviction, the appellate courts might not be so sympathetic. Several legal experts cast doubt on his chances of success, and noted that the case could take years to snake through the courts, all but ensuring he will still be a felon when voters head to the polls in November.

And so, after a five-year investigation and a seven-week trial, Mr. Trump’s New York legal odyssey is only beginning.

list assignment is out of range

The Trump Manhattan Criminal Verdict, Count By Count

Former President Donald J. Trump faced 34 felony charges of falsifying business records, related to the reimbursement of hush money paid to the porn star Stormy Daniels in order to cover up a sex scandal around the 2016 presidential election.

The former president’s supporters are calling on the U.S. Supreme Court to intervene, though that is highly unlikely. In a more likely appeal to a New York court, Mr. Trump would have avenues to attack the conviction, the experts said, but far fewer than he has claimed. The experts noted that the judge whose rulings helped shape the case stripped some of the prosecution’s most precarious arguments and evidence from the trial.

We are having trouble retrieving the article content.

Please enable JavaScript in your browser settings.

Thank you for your patience while we verify access. If you are in Reader mode please exit and  log into  your Times account, or  subscribe  for all of The Times.

Thank you for your patience while we verify access.

Already a subscriber?  Log in .

Want all of The Times?  Subscribe .

Purdue Online Writing Lab Purdue OWL® College of Liberal Arts

Welcome to the Purdue Online Writing Lab

OWL logo

Welcome to the Purdue OWL

This page is brought to you by the OWL at Purdue University. When printing this page, you must include the entire legal notice.

Copyright ©1995-2018 by The Writing Lab & The OWL at Purdue and Purdue University. All rights reserved. This material may not be published, reproduced, broadcast, rewritten, or redistributed without permission. Use of this site constitutes acceptance of our terms and conditions of fair use.

The Online Writing Lab at Purdue University houses writing resources and instructional material, and we provide these as a free service of the Writing Lab at Purdue. Students, members of the community, and users worldwide will find information to assist with many writing projects. Teachers and trainers may use this material for in-class and out-of-class instruction.

The Purdue On-Campus Writing Lab and Purdue Online Writing Lab assist clients in their development as writers—no matter what their skill level—with on-campus consultations, online participation, and community engagement. The Purdue Writing Lab serves the Purdue, West Lafayette, campus and coordinates with local literacy initiatives. The Purdue OWL offers global support through online reference materials and services.

A Message From the Assistant Director of Content Development 

The Purdue OWL® is committed to supporting  students, instructors, and writers by offering a wide range of resources that are developed and revised with them in mind. To do this, the OWL team is always exploring possibilties for a better design, allowing accessibility and user experience to guide our process. As the OWL undergoes some changes, we welcome your feedback and suggestions by email at any time.

Please don't hesitate to contact us via our contact page  if you have any questions or comments.

All the best,

Social Media

Facebook twitter.

U.S. flag

Official websites use .gov

A .gov website belongs to an official government organization in the United States.

Secure .gov websites use HTTPS

A lock ( ) or https:// means you've safely connected to the .gov website. Share sensitive information only on official, secure websites.

How COVID-19 Spreads

COVID-19 spreads when an infected person breathes out droplets and very small particles that contain the virus. These droplets and particles can be breathed in by other people or land on their eyes, noses, or mouth. In some circumstances, they may contaminate surfaces they touch.

Many viruses are constantly changing, including the virus that causes COVID-19. These changes occur over time and can lead to the emergence of variants  that may have new characteristics, including different ways of spreading.

Anyone infected with COVID-19 can spread it, even if they do NOT have symptoms.

Learn more about what you can do to  protect yourself and others .

COVID-19 and Animals

COVID-19 can spread from people to animals in some situations. Pet cats and dogs can sometimes become infected by people with COVID-19. Learn what you should do if you have pets .

Food and Water

There is no evidence to suggest that handling food or consuming food  can spread COVID-19. Follow food safety guidelines when handling and cleaning fresh produce. Do not wash produce with soap, bleach, sanitizer, alcohol, disinfectant, or any other chemical.

Drinking Water

There is also no current evidence that people can get COVID-19 by drinking water. The virus that causes COVID-19 has not been detected in drinking water. Conventional water treatment methods that use filtration and disinfection, such as those in most municipal drinking water systems, should remove or kill the virus that causes COVID-19.​

Natural Bodies of Water (Lakes, Oceans, Rivers)

There are no scientific reports of the virus that causes COVID-19 spreading to people through the water in lakes, oceans, rivers, or other natural bodies of water.

Genetic material from the virus that causes COVID-19 has been found in  untreated wastewater (also referred to as “sewage”). There is no information to date that anyone has become sick with COVID-19 because of direct exposure to treated or untreated wastewater. Wastewater treatment plants use chemical and other disinfection processes to remove and degrade many viruses and bacteria. The virus that causes COVID-19 is inactivated by the disinfection methods used in wastewater treatment.

To receive email updates about COVID-19, enter your email address:

Exit Notification / Disclaimer Policy

  • The Centers for Disease Control and Prevention (CDC) cannot attest to the accuracy of a non-federal website.
  • Linking to a non-federal website does not constitute an endorsement by CDC or any of its employees of the sponsors or the information and products presented on the website.
  • You will be subject to the destination website's privacy policy when you follow the link.
  • CDC is not responsible for Section 508 compliance (accessibility) on other federal or private website.

PLEASE NOTE

  • Range Rover SV
  • MODELS AND SPECIFICATIONS
  • PERSONALIZATION

NEW RANGE ROVER ELECTRIC

DESIGNED AND BUILT IN THE UK

RANGE ROVER  

European Model Shown

Starting at

ELECTRIC HYBRID

Range rover sv, inspiration.

Lead by example. Join the Range Rover Electric waiting list for the opportunity to be among the first to place a pre-order in 2024.

BUILD YOUR RANGE ROVER

The beginning of a new era.

The original luxury SUV. All Range Rover. All electric.

Womans reflection in Range Rover door window

Experience effortless luxury with near-silent fully electric propulsion and access to the fastest public charging networks using Range Rover Electric vehicle 800v electrical architecture.

Peerless all-terrain technology empowers you to climb steeper and wade through upto 33.4 inches of water 22 .

PEERLESS REFINEMENT AND LUXURY

The definition of luxury travel. Range Rover always leads by example, with breathtaking modernity.

New Range Rover tail light parked near sea shore

PIONEERING INNOVATION

Range Rover has a rich bloodline of pioneering innovation and continues to rewrite the rule book for precision and quality.  

Placeholder - Slate Blue

Range Rover extended range plug-in hybrid (PHEV) provides new levels of performance and refinement.

Parked Range Rover charging at the phev station point

An exquisite interpretation of Range Rover luxury and personalization. 

Female sitting in back seat of range rover

Expertly configured by our designers, these curations express the harmony of heritage and innovation synonymous with Range Rover, ready for you to select and order.

ULTIMATE LUXURY (SWB)

ULTIMATE LUXURY (LWB)

INSPIRED (SWB)

INSPIRED (LWB)

list assignment is out of range

ULTIMATE LUXURY

At the pinnacle: the embodiment of luxury.

list assignment is out of range

INSPIRED BY RANGE ROVER DESIGN

Effortless design: crafted by us for your inspiration.

list assignment is out of range

CHOOSE YOUR RANGE ROVER

Start your adventure in a Range Rover

AUTOBIOGRAPHY

MAXIMUM POWER: UP TO 523 HP

MAXIMUM SPEED:  UP TO 155 MPH †   

ACCELERATION 0-60MPH:  UP TO 4.4 SECS ‡†

SEATS: UP TO 7

MAXIMUM POWER:  UP TO 523 HP

MAXIMUM SPEED:  UP TO 155 MPH †

ACCELERATION 0-60MPH:   UP TO 4.4 SECS ‡† 

SEATS:  UP TO 7

MAXIMUM POWER:  UP TO 606 HP

MAXIMUM SPEED:  UP TO 162 MPH †

ACCELERATION 0-60MPH:  UP TO 4.3 SECS ‡†

SEATS:  UP TO 5

THE CHAMPIONSHIPS WIMBLEDON 2024

Range Rover is proud to partner with The Championships, Wimbledon, bringing together our shared passion for refinement and sustainability.

EXPLORE RANGE ROVER

Distinguished design.

Free of superfluous detail. The most desirable Range Rover ever created.

REFINEMENT AND LUXURY

Luxury travel and comfort for up to seven adults.

INTUITIVE TECHNOLOGY

Designed to make your life easier, with a host of driving features.

HEIGHTENED PERFORMANCE

New levels of performance and refinement.

DISCLAIMERS 1-32

  • Stack Overflow Public questions & answers
  • Stack Overflow for Teams Where developers & technologists share private knowledge with coworkers
  • Talent Build your employer brand
  • Advertising Reach developers & technologists worldwide
  • Labs The future of collective knowledge sharing
  • About the company

Collectives™ on Stack Overflow

Find centralized, trusted content and collaborate around the technologies you use most.

Q&A for work

Connect and share knowledge within a single location that is structured and easy to search.

Get early access and see previews of new features.

Python list assignment index out of range

I just can't understand why my list is out of range.

I've been stuck on this problem forever now and can't wrap my head around what the problem might be.

Obviously I'm new into programming.

I get this error message:

Ken Y-N's user avatar

  • How many columns are there in the file? indexList[6] requires every line to have at least 7 fields. –  Barmar Nov 24, 2016 at 3:34
  • 1 unsortedList = [] just creates an unsized list. It looks like you want to use .append(Team(...)) instead. –  h3adache Nov 24, 2016 at 3:34
  • 1 Not related to the question, but I would really not recommend naming a function list , since that is a built-in function in python. –  Omada Nov 24, 2016 at 3:35
  • Why are you using listTeams.readlines(i) to read each line? You already read them all into the list lines . You can just do for tempStr in lines . –  Barmar Nov 24, 2016 at 3:35
  • You can also do teamCount = len(lines) , you don't need to read the file again and count the lines. –  Barmar Nov 24, 2016 at 3:36

2 Answers 2

You can't assign values to List items which don't exist. You can use one of the two methods to solve this problem. One, you can use this command unsortedList.append(Team(indexList[0], indexList[1], indexList[2]). or Second, you can create a list apriori which contains as many zeros as your list will contain by using the command unsortedList= numpy.zeros(i), it will create a list with i number of zeros then you can replace those zeros using your code.

Diksha Dhawan's user avatar

you can't assign to a list element that doesn't already exist, you can use append method

also check the how many items in the array before accessing by index len(indexList) give you the number of items in the array, check that is more than 6 before you get the items 0 to 6 in the indexList

Damith's user avatar

Your Answer

Reminder: Answers generated by artificial intelligence tools are not allowed on Stack Overflow. Learn more

Sign up or log in

Post as a guest.

Required, but never shown

By clicking “Post Your Answer”, you agree to our terms of service and acknowledge you have read our privacy policy .

Not the answer you're looking for? Browse other questions tagged python list indexing or ask your own question .

  • The Overflow Blog
  • Introducing Staging Ground: The private space to get feedback on questions...
  • How to prevent your new chatbot from giving away company secrets
  • Featured on Meta
  • The [tax] tag is being burninated
  • The return of Staging Ground to Stack Overflow
  • The 2024 Developer Survey Is Live
  • Policy: Generative AI (e.g., ChatGPT) is banned

Hot Network Questions

  • What is the U.N. list of shame and how does it affect Israel which was recently added?
  • Transformer with same size symbol meaning
  • Is it legal to deposit a check that says pay to the order of cash
  • Who are the mathematicians interested in the history of mathematics?
  • Integrating slower growing functions to find a faster growing function
  • Why "Power & battery" stuck at spinning circle for 4 hours?
  • How to underline several empty lines
  • A trigonometric equation: how hard could it be?
  • Build the first 6 letters of an Italian codice fiscale (tax identification number)
  • Linear regression: interpret coefficient in terms of percentage change when the outcome variable is a count number
  • Estimating Probability Density for Sample
  • How do I tell which kit lens option is more all-purpose?
  • A man is kidnapped by his future descendants and isolated his whole life to prevent a bad thing; they accidentally undo their own births
  • Death in the saddle
  • Is it true that engines built in Russia are still used to launch American spacecraft?
  • Why does the proposed Lunar Crater Radio Telescope suggest an optimal latitude of 20 degrees North?
  • How do satellites operate below their operating temperature?
  • A phrase that means you are indifferent towards the things you are familiar with?
  • Are your memories part of you?
  • Is it theoretically possible for the sun to go dark?
  • Is it rational for heterosexuals to be proud that they were born heterosexual?
  • Why don't professors seem to use learning strategies like spaced repetition and note-taking?
  • Datasheet recommends driving relay in an uncommon configuration
  • Leaders and Rulers

list assignment is out of range

IMAGES

  1. List Assignment Index Out Of Range Python Indexerror Solution For You

    list assignment is out of range

  2. How To Fix Indexerror List Assignment Index Out Of Range Data

    list assignment is out of range

  3. How to Fix

    list assignment is out of range

  4. IndexError List Assignment Index Out of Range Python

    list assignment is out of range

  5. Python indexerror: list assignment index out of range Solution

    list assignment is out of range

  6. IndexError List Assignment Index Out of Range Solved

    list assignment is out of range

VIDEO

  1. Software Architecture Contact List assignment

  2. OUT RANGE 🥷 || MATCH LOST. (matchlost)🥴😭

  3. POV:The teacher reads your assignment out to the whole class

  4. "Out range" (First ascent)

  5. List Assignment

  6. The Range of Motion Exercise

COMMENTS

  1. Python error: IndexError: list assignment index out of range

    Your list starts out empty because of this: a = [] then you add 2 elements to it, with this code: a.append(3) a.append(7) this makes the size of the list just big enough to hold 2 elements, the two you added, which has an index of 0 and 1 (python lists are 0-based). In your code, further down, you then specify the contents of element j which ...

  2. Python Indexerror: list assignment index out of range Solution

    Python Indexerror: list assignment index out of range Solution. Method 1: Using insert () function. The insert (index, element) function takes two arguments, index and element, and adds a new element at the specified index. Let's see how you can add Mango to the list of fruits on index 1. Python3. fruits = ['Apple', 'Banana', 'Guava']

  3. How to solve the error 'list assignment index out of range' in python

    When your list is empty in python you can not assign value to unassigned index of list. so you have 2 options here:. Use append like : list.append(value); make a loop to assign a value to your list before your main for.Like below: i = 0 while ( i < index_you_want): list[i] = 0 ... #here your main code

  4. IndexError: list assignment index out of range in Python

    # (CSV) IndexError: list index out of range in Python. The Python CSV "IndexError: list index out of range" occurs when we try to access a list at an index out of range, e.g. an empty row in a CSV file. To solve the error, check if the row isn't empty before accessing it at an index, or check if the index exists in the list.

  5. How to fix IndexError: list assignment index out of range in Python

    As you can see, now the 'bird' value is added to the list successfully. Adding the value using the append() method increases the list index range, which enables you to modify the item at the new index using the list assignment syntax.. To summarize, use the append() method when you're adding a new element and increasing the size of the list, and use the list assignment index when you ...

  6. How to Fix the "List index out of range" Error in Python

    The elements in a list are indexed starting from 0. Therefore, to access the first element, do the following: >>> print(x[0]) Our list contains 6 elements, which you can get using the len() built-in function. To access the last element of the list, you might naively try to do the following: >>> print(x[6]) IndexError: list index out of range

  7. Python indexerror: list assignment index out of range Solution

    An index is a value inside an iterable object, such as a list or a string. The message "list assignment index out of range" tells us that we are trying to assign an item to an index that does not exist. In order to use indexing on a list, you need to initialize the list.

  8. How to Fix "IndexError: List Assignment Index Out of Range ...

    How to use the insert() method. Use the insert() method to insert elements at a specific position instead of direct assignment to avoid out-of-range assignments. Example: my_list = [ 10, 20, 30 ] my_list.insert( 3, 987) #Inserting element at index 3 print (my_list) Output: [10, 20, 30, 987] Now one big advantage of using insert() is even if you ...

  9. Python IndexError: List Index Out of Range Error Explained

    IndexError: list index out of range. We can break down the text a little bit. We can see here that the message tells us that the index is out of range. This means that we are trying to access an index item in a Python list that is out of range, meaning that an item doesn't have an index position.

  10. Python List Index Out of Range

    Python Indexerror: list assignment index out of range ExampleIf 'fruits' is a list, fruits=['Apple',' Banan. 3 min read. How to Fix - Indexerror: Single Positional Indexer Is Out-Of-Bounds. While working with Python, many errors occur in Python. IndexError: Single Positional Indexer is Out-Of-Bounds occurs when we are trying to ...

  11. List Index Out of Range

    freeCodeCamp is a donor-supported tax-exempt 501(c)(3) charity organization (United States Federal Tax Identification Number: 82-0779546) Our mission: to help people learn to code for free.

  12. List Index Out of Range

    freeCodeCamp is a donor-supported tax-exempt 501(c)(3) charity organization (United States Federal Tax Identification Number: 82-0779546) Our mission: to help people learn to code for free.

  13. How to Fix Python IndexError: list assignment index out of range

    The reason behind the IndexError: list assignment index out of range in the above code is that we're trying to access the value at the index 3, which is not available in list j. Fix the IndexError: list assignment index out of range in Python. To fix this error, we need to adjust the indexing of iterables in this case list.

  14. Mastering Python's List Assignment: Resolving Index Out of Range Errors

    Solution 1: Validating Indices Before Assignment. One effective solution to prevent "index out of range" errors is to validate the index before attempting to assign a value to it. You can achieve this by checking if the index falls within the list's valid range using the len() function and conditional statements.

  15. 解决:python中出现:list assignment index out of range

    分析:. list assignment index out of range:列表超过限制. 一种情况是:list [index]index超出范围. 另一种情况是:list是一个空的,没有一个元素,进行list [0]就会出现错误!. 解决办法如下:. 将iplist= [] 改为:iplist= {} 白清羽.

  16. List assignment index out of range: Python indexerror solution you

    Solution with Initializing an Array to list assignment Python index out of range. Alternatively, we can initialize our array with some values when we declare it. Because, Tthis will create the index positions at which we can store values inside our "strawberry" array. Therefore, to initialize an array, you can use this code: 1 strawberry ...

  17. IndexError with Django 4.1 #131

    IndexError: list assignment index out of range. Steps to reproduce: Use a MultiSelectField without explicitly specifying max_lenght attribute. Reason for failure is the following django commit: django/django@0ed2919.

  18. IndexError: list assignment index out of range #134

    self.validators[0] = MaxValueMultiFieldValidator(self.max_length) IndexError: list assignment index out of range django==4.1.1 django-multiselectfield==0.1.12

  19. Costs

    Find out how assignment affects what you pay. Clinical laboratory services: $0 for covered clinical laboratory services. Home health care: $0 for covered home health care services. 20% of the Medicare-approved amount for durable medical equipment (like wheelchairs, walkers, hospital beds, and other equipment).

  20. What Part B covers

    Part B covers 2 types of services. Medically necessary services: Services or supplies that are needed to diagnose or treat your medical condition and that meet accepted standards of medical practice. Preventive services: Health care to prevent illness (like the flu) or detect it at an early stage, when treatment is most likely to work best.; You pay nothing for most preventive services if you ...

  21. CDC Current Outbreak List

    Multistate Foodborne Outbreaks - Foodborne outbreaks listed by year; Hepatitis A Outbreaks - Hepatitis A outbreak investigations since 2013 where CDC supported or led the investigation.; US Outbreaks Linked to Contact with Animals or Animal Products; Health Alert Network - Health alerts, health advisories, updates, and info service messages. Designed for public health and medical ...

  22. Trump Will Appeal Conviction, but Has Few Ways to Overturn Decision

    The judge in Donald J. Trump's case closed off many avenues of appeal, experts said, though his lawyers might challenge the novel theory at the case's center. By Ben Protess, William K ...

  23. Welcome to the Purdue Online Writing Lab

    Teachers and trainers may use this material for in-class and out-of-class instruction. Mission The Purdue On-Campus Writing Lab and Purdue Online Writing Lab assist clients in their development as writers—no matter what their skill level—with on-campus consultations, online participation, and community engagement.

  24. List Assignment Out of Range

    0. The issue is that you checked the length of my_list and insert values to new_list, the new list is empty but you insert a value to index 1. What you can do is deep copy my_list and modify the new list. import copy. and then replace new_list=[] with new_list = copy.deepcopy(my_list) then the new_list result would be.

  25. How Coronavirus Spreads

    COVID-19 spreads when an infected person breathes out droplets and very small particles that contain the virus. These droplets and particles can be breathed in by other people or land on their eyes, noses, or mouth. In some circumstances, they may contaminate surfaces they touch.

  26. Luxury SUV

    Experience effortless luxury with near-silent fully electric propulsion and access to the fastest public charging networks using Range Rover Electric vehicle 800v electrical architecture. Peerless all-terrain technology empowers you to climb steeper and wade through upto 33.4 inches of water 22 .

  27. Why does this iterative list-growing code give IndexError: list

    j is an empty list, but you're attempting to write to element [0] in the first iteration, which doesn't exist yet. Try the following instead, to add a new element to the end of the list: for l in i: j.append(l) Of course, you'd never do this in practice if all you wanted to do was to copy an existing list. You'd just do: j = list(i)

  28. Marlins' Edwards gets chance to play natural position of shortstop

    After completing a rehab assignment last month, Edwards saw action in 13 games for Jacksonville, hitting .365 with an OPS of .892 and OBP of .411, and playing shortstop.

  29. Adobe Creative Cloud for students and teachers

    Students and Teachers. Introductory Pricing Terms and Conditions Creative Cloud Introductory Pricing Eligible students 13 and older and teachers can purchase an annual membership to Adobe® Creative Cloud™ for a reduced price of for the first year. At the end of your offer term, your subscription will be automatically billed at the standard subscription rate, currently at (plus applicable ...

  30. Python list assignment index out of range

    1. You can't assign values to List items which don't exist. You can use one of the two methods to solve this problem. One, you can use this command unsortedList.append (Team (indexList [0], indexList [1], indexList [2]). or Second, you can create a list apriori which contains as many zeros as your list will contain by using the command ...